subject
Mathematics, 02.03.2022 16:50 mermer11

Asset W has an expected return of 13.75 percent and a beta of 1.4. If the risk-free rate is 4.65 percent, complete the following table for portfolios of Asset W and a risk-free asset. (Leave no cells blank - be certain to enter "0" wherever required. Do not round intermediate calculations. Enter your portfolio expected return answers as a percent rounded to 2 decimal places, e. g., 32.16. Enter your portfolio beta answers rounded to 3 decimal places, e. g., 32.161.)

ansver
Answers: 3

Another question on Mathematics

question
Mathematics, 21.06.2019 14:50
What is the inverse of the function f(x) = 2x + 1?
Answers: 1
question
Mathematics, 21.06.2019 17:30
If you apply the below transformations to the square root parent function, f(x) = vx, what is the equation of the new function? • shift 12 units right. • shift seven units down.
Answers: 1
question
Mathematics, 21.06.2019 17:30
1mile equals approximately 1.6 kilometers. which best approximates the number of miles in 6 kilometers?
Answers: 1
question
Mathematics, 21.06.2019 18:30
Iam a rectangle. my perimeter is 60 feet. how much are do i cover?
Answers: 1
You know the right answer?
Asset W has an expected return of 13.75 percent and a beta of 1.4. If the risk-free rate is 4.65 per...
Questions
question
Mathematics, 02.09.2019 03:30
question
Mathematics, 02.09.2019 03:30
question
History, 02.09.2019 03:30
question
Mathematics, 02.09.2019 03:30